Đến nội dung

davidsilva98

davidsilva98

Đăng ký: 29-11-2013
Offline Đăng nhập: Riêng tư
****-

#668305 $p \geq q+2$

Gửi bởi davidsilva98 trong 14-01-2017 - 20:37

Bài 10. Cho là một số nguyên dương thỏa mãn + 1 = ≥ là số nguyên tố.
Gọi là ước số nguyên tố lớn nhất của 
$b^{y}$ + 1là ước số nguyên tố nhỏ nhất của

y. Chứng minh rằng:
≥ + 2.

 

Dễ thấy nếu $q=2$ thì bài toán được giải quyết. Do đó xét $q>2$ nên $y$ lẻ

 

Gọi $t=ord_{p}(q-1)$. Ta xét các trường hợp sau:

 

Trường hợp 1. Nếu $t=1$ thì $ord_{p}(q-1)=1$. Dẫn đến $$q-1\equiv 1(mod\: p)\Rightarrow \left.\begin{matrix}p \end{matrix}\right|q-2\Rightarrow q-2\geq p\:(1)$$ Mặt khác, ta lại có $\left.\begin{matrix}q \end{matrix}\right|(q-1)^y+1$ và  $p$ là ước số nguyên tố lớn nhất của $(q-1)^y+1$ nên suy ra $q\leq p\:(2)$

 

Từ $(1)$ và $(2)$ ta được $q\leq p\leq q-2$. Dẫn đến điều mâu thuẫn.


Trường hợp 2. Nếu $t=2$ thì $ord_{p}(q-1)=2\Rightarrow \left.\begin{matrix}p \end{matrix}\right|(q-1)^2-1=q(q-2)$

 

Do $p,q$ là các số nguyên tố nên $(p,q)=1$. Dẫn đến $ \left.\begin{matrix}p \end{matrix}\right|q-2\Rightarrow q-2\geq p\:(3)$

 

Mặt khác, ta lại có $\left.\begin{matrix}q \end{matrix}\right|(q-1)^y+1$ và  $p$ là ước số nguyên tố lớn nhất của $(q-1)^y+1$ nên suy ra $q\leq p\:(4)$

 

Từ $(3)$ và $(4)$ ta được $q\leq p\leq q-2$. Dẫn đến điều mâu thuẫn.

 

Trường hợp 3. Xét $t>2$.Từ giả thiết suy ra $$\left.\begin{matrix}p \end{matrix}\right|(q-1)^{2y}-1\Rightarrow \left.\begin{matrix}t\end{matrix}\right|2y$$ Do $t>2$ nên $ \left.\begin{matrix}t \end{matrix}\right|y$ mà $q$ là ước số nguyên tố nhỏ nhất của $y$. Dẫn đến $q\leq t(*)$

 

Mặt khác, theo định lý Fermat nhỏ ta được $$(q-1)^{p-1}\equiv 1(mod\: p)$$ Từ đó suy ra $t\leq p-1(**)$. Kết hợp giữa $(*)$ và $(**)$ suy ra $q\leq p-1\Leftrightarrow p\geq q+1$

 

Do $p,q$ là số nguyên tố lẻ nên $p\neq q+1$ nên $p\geq q+2$

 




#651307 Cmr:$\frac{b}{a}+\frac{c}{b...

Gửi bởi davidsilva98 trong 26-08-2016 - 00:36

Cho các số thực a,b,c thỏa mãn $a+b+c\geq \frac{1}{a}+\frac{1}{b}+\frac{1}{c}$.Cmr:

$\frac{b}{a}+\frac{c}{b}+\frac{a}{c}\geq \frac{1}{ab}+\frac{1}{bc}+\frac{1}{ca}$

 

Theo bất đẳng thức Cauchy-Schwarz ta được $$\frac{b}{a}+\frac{c}{b}+\frac{a}{c}=\frac{b^2}{ab}+\frac{c^2}{bc}+\frac{a^2}{ca}\geq \frac{(a+b+c)^2}{ab+bc+ca}$$ Mặt khác, theo giả thiết ta suy ra $$abc(a+b+c)\geq ab+bc+ca$$ Từ đó ta suy ra $$\frac{b}{a}+\frac{c}{b}+\frac{a}{c}\geq \frac{(a+b+c)^2}{abc(a+b+c)}=\frac{a+b+c}{abc}$$ Vậy bất đẳng thức được chứng minh.




#651230 $\left\{\begin{matrix} x^2 y=x+z \\ y^2 z=y+x...

Gửi bởi davidsilva98 trong 25-08-2016 - 17:57

Bài số $\fbox{2a}$:

Giải hệ phương trình:

$\left\{\begin{matrix} x^2 y=x+z \\ y^2 z=y+x \\ z^2 x = z + y\\ \end{matrix}\right.$
 

 

Hệ phương trình trên tương đương với $$\left\{\begin{matrix} y(x^2+1)=x+y+z\; (1) \\ z(y^2+1)=x+y+z \;  (2)\\ x(z^2+1) = x+y+z\; (3)\\ \end{matrix}\right.$$ Nếu $x+y+z=0$ thì ta suy ra $x=y=z=0$

 

Nếu $x+y+z\neq 0$ thì dẫn đến $x,y,z$ cùng dấu. Ta có nhận xét $(x_{0};y_{0};z_{0})$ là nghiệm của hệ thì $(-x_{0};-y_{0};-z_{0})$ cũng là nghiệm của hệ. Do đó, ta chỉ cần xét $x,y,z>0$.

 

Trường hợp 1. Nếu $x>y$ thì dẫn đến $$x^2+1>y^2+1\Rightarrow y<z\Rightarrow y^2+1<z^2+1\Rightarrow z>x\Rightarrow z^2+1>x^2+1\Rightarrow x<y$$ Do đó dẫn đến vô lý.

 

Trường hợp 2. Nếu $x<y$ thì tương tự ta có được $x>y$. Dẫn đến mâu thuẫn

 

Suy ra $x=y=z$. Khi đó ta có phương trình $x^3=2x\Leftrightarrow x=0\vee x=\pm \sqrt{2}$

 

Vậy nghiệm của hệ là $\left ( x,y,z \right )=\left \{ \left ( 0,0,0 \right ) ;\left ( \sqrt{2},\sqrt{2},\sqrt{2} \right );\left ( -\sqrt{2},-\sqrt{2},-\sqrt{2} \right )\right \}$

 

 




#650741 3 bất đẳng thức không thể cùng đúng

Gửi bởi davidsilva98 trong 22-08-2016 - 03:18

Cho a, b, c, x, y, z thỏa mãn $a> 0$ và $(b-1)^{2}-4ac<0$. Chứng minh rằng 3 bất đẳng thức $ax^{2}+bx+c\leq y$, $ay^{2}+by+c\leq z$, $az^{2}+bz+c\leq x$ không thể cùng đúng.

 

Giả sử cả $3$ bất đẳng thức trên cùng đúng. Khi đó, cộng $3$ bất đẳng thức ta được $$a(x^2+y^2+z^2)+(b-1)(x+y+z)+3c\leq 0$$ Mặt khác, ta lại có bất đẳng thức sau $$x^2+y^2+z^2\geq \frac{(x+y+z)^2}{3}$$ Từ đó suy ra $$\frac{a(x+y+z)^2}{3}+(b-1)(x+y+z)+3c\leq 0$$ Đặt $t=\frac{x+y+z}{3}$. Dẫn đến $$\frac{at^2}{3}+(b-1)t+3c\leq 0\Leftrightarrow \frac{a}{3}\left [t+\frac{3(b-1)}{2a}  \right ]^2+\frac{3\left [ 4ac-(b-1)^2 \right ]}{4a}\leq 0$$ Kết hợp giả thiết ta thấy bất đẳng thức trên vô lý. Dẫn đến điều giả sử là sai

 
Vậy bài toán được chứng minh.



#650584 Đề thi chọn đội tuyển tỉnh Bình Dương năm 2016

Gửi bởi davidsilva98 trong 21-08-2016 - 02:07

Câu 3b. Trên mặt phẳng cho $2017$ điểm sao cho với $3$ điểm bất kì trong số các điểm đó ta luôn tìm được $2$ điểm để đoạn thẳng được tạo thành có độ dài bé hơn $1$. Chứng minh rằng luôn tồn tại một đường tròn bán kính bằng $1$ chứa không ít hơn $1009$ điểm đã cho.

 

Lời giải. 

Gọi $A_{1},A_{2},..,A_{2017}$ là các điểm đã cho trên mặt phẳng.

 

* Nếu tất cả các đoạn thẳng được tạo ra từ $2$ điểm trong $2017$ điểm đã cho có độ dài bé hơn $1$. Như vậy các điểm đã cho nằm trong đường tròn tâm $A_{1}$ bán kính bằng $1$.

 

* Nếu tồn tại đoạn thẳng có độ dài lớn hơn hoặc bằng $1$. Giả sử $A_{1}A_{2}$ là đoạn thẳng có độ dài lớn nhất. Khi đó ta xét $3$ điểm $A_{1},A_{2},A_{k},\: k=\overline{3,2017}$. 

 

Do $A_{1}A_{2}$ có độ dài lớn hơn hoặc bằng $1$ nên một trong hai đoạn thẳng $A_{1}A_{k}$ và $A_{2}A_{k}$ có độ dài nhỏ hơn $1$.

 

Gọi $B_{1}$ là số đoạn thẳng $A_{1}A_{k}$,  có độ dài nhỏ hơn $1$

      $B_{2}$ là số đoạn thẳng $A_{2}A_{k}$,  có độ dài nhỏ hơn $1$

 

Theo nguyên lý Đirichlet, tồn tại một trong hai số $B_{1},B_{2}$ lớn hơn hoặc bằng $1008$. Giả sử là $B_{1}$.

 

Như vậy tồn tại ít nhất $1009$ điểm trong đường tròn tâm $A_{1}$ bán kính bằng $1$ kể cả điểm $A_{1}$

 

 




#650583 Đề thi chọn đội tuyển tỉnh Bình Dương năm 2016

Gửi bởi davidsilva98 trong 21-08-2016 - 01:26

Câu 4. Cho $x,y,z$ là $3$ số thực dương thỏa $$x^3+y^2+z=2\sqrt{3}+1$$ Tìm giá trị nhỏ nhất của biểu thức $$P=\frac{1}{x}+\frac{1}{y^2}+\frac{1}{z^3}$$ Lời giải.

Áp dụng bất đẳng thức AM-GM ta được: $$\frac{x^3}{3}+\frac{1}{3x}+\frac{1}{3x}+\frac{1}{3x}\geq \frac{4}{3}$$ $$\frac{y^2}{3}+\frac{1}{y^2}\geq \frac{2}{\sqrt{3}}$$ $$\frac{z}{9}+\frac{z}{9}+\frac{z}{9}+\frac{1}{z^3}\geq \frac{4\sqrt{3}}{9}$$ Từ đó suy ra $$P=\frac{1}{x}+\frac{1}{y^2}+\frac{1}{z^3}\geq \frac{12+10\sqrt{3}}{9}-\frac{x^3+y^2+z}{3}=\frac{9+4\sqrt{3}}{9}$$ Đẳng thức xảy ra khi $x=1;\: y=\sqrt[4]{3};\: z=\sqrt{3}$




#626505 Cho $p\in \mathbb{P}$. Chứng minh luôn tồn tại...

Gửi bởi davidsilva98 trong 11-04-2016 - 00:48

Chứng minh rằng nếu hai số nguyên dương $a, b$ thỏa $p = a + b$ với $p$ là số nguyên tố thì $2ab = c^{2} + d^{2}$

Nhận xét

 

Theo đề bài của bạn thì với $a=2$ và $b=3$ thì không tồn tại $c,d$. Mình nghĩ đề như sau mới chính xác:  

 

Chứng minh nếu hai số nguyên dương $a,b$ thỏa $a+b=p$ với $p$ là số nguyên tố thì tồn tại $c,d$ sao cho $2ab=c^{2}-d^{2}$

 

Chứng minh như sau: Với $p=2$ ta dễ chứng minh được. Xét $p> 2$ thì $p$ lẻ nên có thể giả sử $a$ chẵn.

 

Khi đó ta có thể chọn $c=\frac{a}{2}+b$ và $d=\frac{a}{2}-b$ thì $2ab=c^2-d^2$.

 

Bài toán được chứng minh.




#611224 $\sqrt[3]{(x+y)(y+z)(z+x)}\geq \frac{2...

Gửi bởi davidsilva98 trong 27-01-2016 - 00:57

Cho $x,y,z$ là các số thực dương. Chứng minh rằng:

$\sqrt[3]{(x+y)(y+z)(z+x)}\geq \frac{2}{\sqrt{3}}\sqrt{xy+yz+zx}$

 

Ta chuẩn hóa $xy+yz+zx=3$. Ta chỉ cần chứng minh $$(x+y)(y+z)(z+x)\geq 8$$

Thật vậy: $$(x+y)(y+z)(z+x)\geq 8\Leftrightarrow (x+y+z)(xy+yz+zx)-xyz\geq 8\Leftrightarrow 3(x+y+z)-xyz\geq 8$$

Bất đẳng thức trên đúng do $xy+yz+zx=3$ nên $x+y+z\geq 3,\; xyz\leq 1$




#610799 $\frac{ab}{c+ab}+\frac{bc}{...

Gửi bởi davidsilva98 trong 24-01-2016 - 20:04

Cho a,b,c>0 va a+b+c=1 chung minh $\frac{ab}{c+ab}+\frac{bc}{a+bc}+\frac{ca}{b+ac}\geq \frac{3}{4}$

 

Bất đẳng thức cần chứng minh tương đương $$\sum \frac{ab}{c+ab}\geq \frac{3}{4}\Leftrightarrow \sum \frac{ab}{c(a+b+c)+ab}\geq \frac{3}{4}\Leftrightarrow \sum \frac{ab}{(c+a)(c+b)}\geq \frac{3}{4}$$

$$\Leftrightarrow 4\sum ab(a+b)\geq 3(a+b)(b+c)(c+a)\Leftrightarrow 4\sum ab(a+b)\geq 3\left ( \sum ab(a+b) +2abc\right )$$

$$\Leftrightarrow \sum ab(a+b)\geq 6abc\Leftrightarrow \sum a(b-c)^{2}\geq 0$$

Vậy bất đẳng thức được chứng minh.




#598578 Chọn đội tuyển dự thi VMO 2016 tỉnh Đồng Nai

Gửi bởi davidsilva98 trong 16-11-2015 - 00:15

Buổi thi thứ hai.

 
Bài 5. (7 điểm)
Cho tam giác $ABC$ có $AB> AC$. Gọi $M,N,P$ lần lượt là trung điểm $BC,CA,AB$. Các đường trung trực của $AB,AC$ cắt tia $AM$ tại $D,E$ tương ứng. Đường thằng $BD$ và $CE$ cắt nhau tại $F$ ($F$ nằm trong tam giác $ABC$)
 
       a) Chứng minh $AF$ là phân giác ngoài góc $EFD$
     
       b) Chứng minh rằng $A,N,F,P$ cùng nằm trên một đường tròn.

 

Duới đây mình xin trình bày một cách giải ảo diệu câu a  :D

Đồng Nai 2016.PNG

Ta có: $$\frac{FE}{FD}=\frac{sin\widehat{EDF}}{sin\widehat{DEF}}=\frac{sin2\widehat{BAM}}{sin2\widehat{CAM}}=\frac{sin\widehat{BAM}}{sin\widehat{CAM}}.\frac{cos\widehat{BAM}}{cos\widehat{CAM}}=\frac{sin\widehat{BAM}}{sin\widehat{CAM}}.\frac{AP/AD}{AN/AE}$$ 

$$\Rightarrow \frac{FE}{FD}=\frac{sin\widehat{BAM}}{sin\widehat{CAM}}.\frac{AB}{AC}.\frac{AE}{AD}=\frac{AE}{AD}.\frac{\frac{1}{2}.AB.AM.sin\widehat{BAM}}{\frac{1}{2}.AC.AM.sin\widehat{CAM}}=\frac{AE}{AD}.\frac{S_{ABM}}{S_{ACM}}=\frac{AE}{AD}$$ 

Suy ra $AF$ là phân giác ngoài góc $DFE$




#598333 Chọn đội tuyển dự thi VMO 2016 tỉnh Đồng Nai

Gửi bởi davidsilva98 trong 14-11-2015 - 21:32

Buổi thi thứ nhất.

 

Bài 1. (5 điểm)

Cho dãy số $\left ( x_{n} \right )$ được xác định bởi $$\left\{\begin{matrix} x_{1}\in \left ( 0;1 \right )\\ x_{n+1}=x_{n}+\left ( \frac{x_{n}}{n} \right )^{2}, \forall n\geq 1\end{matrix}\right.$$ Dãy số $\left ( x_{n} \right )$ có hội tụ không? Tại sao?

 
Bài 2. (5 điểm)
Tìm tất cả các hàm số liên tục $f:R\rightarrow R$ thỏa: $$f\left ( 2x-y \right )-6\left ( x+1 \right )\left ( x-y \right )^{2}=2f\left ( x \right )-f\left ( y \right ),\: \forall x,y\in R$$
 
Bài 3. (5 điểm)
Cho bảng $3\times 3$ (gồm 3 hàng ngang và 3 cột dọc). Kí hiệu ô vuông $\left ( i;j \right )$ là ô vuông giao của hàng thứ $i$ (tính từ trên xuống) và cột thứ $j$ (tính từ trái sang phải). Có bao nhiêu cách điền vào các ô vuông, mỗi ô một số tự nhiên (không nhất thiết phân biệt) sao cho tổng mỗi hàng và tổng mỗi cột đều bằng $2015$, đồng thời trong các ô $\left ( i;i \right ),\:  i=\overline{1,3}$ thì ô $\left ( 2;2 \right )$ ghi số nhỏ nhất.
 
Bài 4. (5 điểm)
Cho tam giác $ABC$. đường tròn nội tiếp $I$ tiếp xúc với hai cạnh $AB,AC$ lần lượt tại $E,F$. Lấy điểm $J$ thuộc đường thẳng $EF$ sao cho $BJ$ song song $AC$. Gọi $K$ là giao điểm của $CJ$ và $AB$. Chứng minh rằng $IK$ song song $EF$.
 
 
Buổi thi thứ hai.
 
Bài 5. (7 điểm)
Cho tam giác $ABC$ có $AB> AC$. Gọi $M,N,P$ lần lượt là trung điểm $BC,CA,AB$. Các đường trung trực của $AB,AC$ cắt tia $AM$ tại $D,E$ tương ứng. Đường thằng $BD$ và $CE$ cắt nhau tại $F$ ($F$ nằm trong tam giác $ABC$)
 
       a) Chứng minh $AF$ là phân giác ngoài góc $EFD$
     
       b) Chứng minh rằng $A,N,F,P$ cùng nằm trên một đường tròn.
 
Bài 6. (7 điểm)
Cho hai số nguyên dương $a,b$ sao cho $a^{2}+2b$ là số chính phương. Chứng minh rằng $a^{2}+b$ có thể phân tích thành tổng của hai số chính phương.
 
Bài 7. (6 điểm)
Trong dịp chào mừng ngày nhà giáo Việt Nam $20-11$ và khánh thành trường mới, Đoàn trường THPT Chuyên Lương Thế VInh tổ chức thi văn nghệ. Để chọn ra các tiết mục xuất sắc biểu diễn trong ngày lễ, Ban chấp hành Đoàn trường đã tổ chức duyệt văn nghệ trong $6$ buổi. Biết rằng trong mỗi buổi có đúng $100$ học sinh tham gia để cổ vũ cho các tiết mục và không có hai học sinh nào mà hợp lại tham gia đủ cả $6$ buổi. Hỏi có ít nhất bao nhiêu học sinh đã tham gia cổ vũ cho các tiết mục văn nghệ trong $6$ buổi trên.



#561674 ĐỀ THI OLYMPIC CHUYÊN KHOA HỌC TỰ NHIÊN NĂM 2015

Gửi bởi davidsilva98 trong 26-05-2015 - 14:21

 

Ngày thi thứ nhất

 

Câu I: Tìm tất cả các số nguyên tố $p$ sao cho $3^p+4^p$ là số chính phương

 

 

Đặt $3^{p}+4^{p}=k^{2}\; (k\in \mathbb{N})$

 

Dễ thấy $p=2$ thỏa mãn đề bài và $p=7$ không thỏa.

 

Xét trường hợp $p\neq 7$. Do $p$ lẻ nên $3^{p}+4^{p}\: \vdots \: 7\Rightarrow k=7^{a}.b$ với $(b,7)=1$

 

Theo định lý LTE ta được $$2a=v_{7}(3^{p}+4^{p})=v_{7}(3+4)+v_{7}(p)=1$$ Điều này vô lý do $a\in \mathbb{N}$. Vậy $p=2$




#559991 $x^{7}+y^{7}=1998^{z}$

Gửi bởi davidsilva98 trong 17-05-2015 - 16:21

Bài toán. Chứng minh rằng phương trình $$x^{7}+y^{7}=1998^{z}$$ không có nghiệm nguyên dương.




#559107 Chứng minh $\frac{a^2+b^2}{ab+1}$là số chí...

Gửi bởi davidsilva98 trong 13-05-2015 - 14:36

Cho a,b là các số nguyên dương thỏa mãn $a^2+b^2$ chia hết cho $ab+1$

 

Chứng minh $\frac{a^2+b^2}{ab+1}$là số chính phương

 

Cho $k$ cố định. Xét tập sau $$S=\left \{ (a,b)\in \mathbb{N}^{2} :\frac{a^{2}+b^{2}}{ab+1}=k\right \}$$

Gọi $(a_{0},b_{0})\in S$ và $a_{0}+b_{0}$ nhỏ nhất. Do $a,b$ có vai trò như nhau nên giả sử $a_{0}\geq b_{0}$
 
Xét phương trình bậc hai ẩn $X$:  $$\frac{X^{2}+b_{0}^{2}}{Xb_{0}+1}=k\Leftrightarrow X^{2}-kb_{0}X+b_{0}^{2}-k=0\; (1)$$
Phương trình $(1)$ có một nghiệm là $a_{0}$ và gọi nghiệm còn lại là $a_{1}$.
 
Theo định lý Viete ta có $a_{0}+a_{1}=kb_{0}$ và $a_{0}.a_{1}=b_{0}^{2}-k$
 
Do $a_{0}\in Z^{+}$ nên $a_{1}\in Z$. Ta xét các trường hợp của $a_{1}$.
 
$\cdot \; a_{1}< 0\rightarrow 0=a_{1}^{2}-ka_{1}b_{0}+b_{0}^{2}-k\geq a_{1}^{2}+k+b_{0}^{2}-k$. Điều này vô lý.
 
$\cdot \; a_{1}>0\rightarrow (a_{1},b_{0})\in S\rightarrow a_{1}\geq a_{0}\rightarrow a_{0}^{2}\leq a_{0}a_{1}=b_{0}^{2}-k< b_{0}^{2}\rightarrow a_{0}< b_{0}$. Điều này mâu thuẫn với điều giả sử
 
$\cdot \; a_{1}=0\rightarrow b_{0}^{2}-k=0\rightarrow k=b_{0}^{2}$. Vậy chứng minh hoàn thành.



#558914 cho tam giác ABC cân tại A

Gửi bởi davidsilva98 trong 12-05-2015 - 15:59

Qua C dựng đt d vuông góc với AC. Từ B dựng BE vuông góc với d. Gọi M là trung điểm BE. AM giao với CE tại D. CMR BC vuông góc với BD

 

Gọi $F$ là giao điểm của $AC$ và $BD$. Xét tam giác $BFC$ có $M$ là trung điểm $BE$ nên $A$ là trung điểm $CF$

 

Suy ra $AF=AC=AB$ (do tam giác $ABC$ cân ở $A$) nên $\angle CBF=90^{0}\Rightarrow \angle CBD=90^{0}$

 

Dẫn đến $BC$ vuông góc $BD$